Subscription pricing
In the 7Sage version of PT33 and in the Logic Games Bundle,
PT33.S4.Q14 (LG) says:
"most be true"
I paused for like three seconds figuring out what it means before realizing that this is a typo.
I think I found another one the other day, but I can't remember which PT it was from.
So please let me know if you find any typos so that @"7Sage Admin" can fix someday...
0
8 comments
@7sagestudentservices said:
@akikookmt881 said:
@7sagestudentservices
I should probably email you, but the PT30.S2.Q13 on the e-doc of the "Flaw Or Descriptive Weakening Drill" of the "LR Drills - Preptest 30 to 39" (https://classic.7sage.com/lesson/lr-drills-preptest-30-to-39/) does not show the latter half of the answer choice (C).
Maybe this is only happening to me, but could you maybe check this?
http://imgur.com/9TFx0qH
Weird, it shows for me. You may want to clear your cache:
http://classic.7sage.com/clear-cache/
Weird! Thank you, I'll try!!!
@akikookmt881 said:
@7sagestudentservices
I should probably email you, but the PT30.S2.Q13 on the e-doc of the "Flaw Or Descriptive Weakening Drill" of the "LR Drills - Preptest 30 to 39" (https://classic.7sage.com/lesson/lr-drills-preptest-30-to-39/) does not show the latter half of the answer choice (C).
Maybe this is only happening to me, but could you maybe check this?
http://imgur.com/9TFx0qH
Weird, it shows for me. You may want to clear your cache:
http://classic.7sage.com/clear-cache/
@7sagestudentservices
I should probably email you, but the PT30.S2.Q13 on the e-doc of the "Flaw Or Descriptive Weakening Drill" of the "LR Drills - Preptest 30 to 39" (https://classic.7sage.com/lesson/lr-drills-preptest-30-to-39/) does not show the latter half of the answer choice (C).
Maybe this is only happening to me, but could you maybe check this?
http://imgur.com/9TFx0qH
@jy-ping said:
Yup, that's a typo. Will be getting that fixed.
I found another typo! (tagging @7sagestudentservices, @7sagestudentservices as well)
PT30.S1. Game 3
The fourth rule says:
http://imgur.com/C79C05W
It should be "Neither O's nor T's."
Yup, that's a typo. Will be getting that fixed.
Oh damn! I'm looking at Kaplan's PDF of the test.
@gregoryalexanderdevine723 said:
Interestingly enough, that typo was on the original PT. There are definitely a few more I've seen too. mostly from the older tests.
Hmmm....Which version are you looking at? On the page 178 of my "The Next 10 Actual, Official LSAT PrepTests" (https://www.amazon.com/Next-Actual-Official-LSAT-PrepTests/dp/0979305055 ), the Question 14 says "must be true."
Interestingly enough, that typo was on the original PT. There are definitely a few more I've seen too. mostly from the older tests.